LSAT and Law School Admissions Forum

Get expert LSAT preparation and law school admissions advice from PowerScore Test Preparation.

 ellenb
  • Posts: 260
  • Joined: Oct 22, 2012
|
#9683
Dear Powerscore,

I just want to know why answer choice A is the correct one. I picked this one, but still want to make sure that I know that my thought process is correct.

Thanks
Ellen
 Steve Stein
PowerScore Staff
  • PowerScore Staff
  • Posts: 1153
  • Joined: Apr 11, 2011
|
#9684
Hi Ellen,

It's great that you got that one right; it might be more helpful if you told us a bit more about your thought process; that way we can confirm that you got the right answer for exactly the right reason--thanks!

~Steve
 ellenb
  • Posts: 260
  • Joined: Oct 22, 2012
|
#9686
Thanks Steve,

I was thinking along the lines such as if there was not an actual increase than the event did not occur, for example people who are trying to lose weight,
if one the scale they still have the same 150 showing, and exercising someone might say that that there has been no change. However, they might be putting muscle weight and have less fat. Does it make sense?

so because there is no evident consequence, the change did not occur. Because their bank accounts stayed the same the argument assumes that there was economical change. (they might have wanted to spend more and did not).


Thanks

Ellen
 Luke Haqq
PowerScore Staff
  • PowerScore Staff
  • Posts: 747
  • Joined: Apr 26, 2012
|
#9700
Hi Ellen,

Your reasoning makes sense to me. One way to see the passage proceeds is by noting the cause-and-effect relationship in it. Answer choice A, as your analogy about weight rightly suggests, is correct because it's correctly describes how the stimulus proceeds, but concluding that because there is no effect, the cause cannot have occurred.

Since you also had a question about #18, is worth noting that one could take this to be a flaw in the argument. Just because their saving accounts are not growing does not guarantee that people with jobs have not diminished their spending. For example, they may be paying off debts accelerated rate, in which case their savings accounts may not grow while their spending may still decrease.
 LSAT2018
  • Posts: 242
  • Joined: Jan 10, 2018
|
#48092
I would just like to clarify the answer here. When it says 'concluding that since an expected consequence of a supposed development did not occur, that development itself did not take place,' the expected consequence refers to decreased spending, and the supposed development refers to the actual causal argument that staff reduction leads to decreased spending?
 Adam Tyson
PowerScore Staff
  • PowerScore Staff
  • Posts: 5153
  • Joined: Apr 14, 2011
|
#48179
The expected consequence that did not take place is increased savings, and the development that the author thinks did not take place is reduced spending. In other words, the author assumes that if spending went down, savings would have gone up, and since savings didn't go up, spending must therefore not have gone down.
User avatar
 LeslyLSATinLA
  • Posts: 16
  • Joined: Jan 30, 2023
|
#99411
I picked E because I thought it was more stronger than answer choice A.

I wrote down the following:

staff reductions--> people cutting back on buying stuff
spending less--> not putting money in savings accounts.


I noticed that there was no effect and no cause, so it led me to E. Can someone explain why it is not E?
 Adam Tyson
PowerScore Staff
  • PowerScore Staff
  • Posts: 5153
  • Joined: Apr 14, 2011
|
#99441
E is incorrect because it incorrectly describes the nature of the evidence, Lesley. That answer says the evidence is "ambiguous," but in fact the evidence is crystal clear: there has been no unusual increase in savings accounts.

Another reason to eliminate answer E is that it incorrectly describes the conclusion. In the stimulus, the conclusion was completely certain: "actual spending by such people is undiminished." There is no element of probability here. But answer E says the conclusion is that a change in spending is "most likely" not happening. That doesn't match the strength of the author's claim, and so it fails to describe how they made their argument.

For an answer like E to be correct, the argument would have to have been more like this:

"The evidence in this case could support that claim, or it could undermine it, depending on how you interpret it. Therefore, the claim is probably false."

Make sure that when you select an answer to a Method of Reasoning question, the answer accurately describes the argument in every way! If the type of evidence is different, or if the strength of the language is different, it's a loser.

Get the most out of your LSAT Prep Plus subscription.

Analyze and track your performance with our Testing and Analytics Package.